Trắc nghiệm Giải tích 12 Chương 2 Bài 6 Bất phương trình mũ và bất phương trình lôgarit

  1. Tác giả: LTTK CTV
    Đánh giá: ✪ ✪ ✪ ✪ ✪

    • Câu 1:
      Giải bất phương trình \({9^x} - {\log _2}8 < {2.3^x}.\)
      • A. x>0
      • B. x<0
      • C. x>1
      • D. x<1
    • Câu 2:
      Tìm tập nghiệm S của bất phương trình: \({2^{{x^2} - x + 1}} > {4^{x + 1}}.\)
      • A. \(S = \left( {\frac{{3 - \sqrt {13} }}{2};\frac{{3 + \sqrt {13} }}{2}} \right)\)
      • B. \(S = \left( { - \infty ;\frac{{3 - \sqrt {13} }}{2}} \right) \cup \left( {\frac{{3 + \sqrt {13} }}{2}; + \infty } \right)\)
      • C. \(S = \left( { - \infty ; - \sqrt 2 } \right) \cup \left( {\sqrt 2 ; + \infty } \right)\)
      • D. \(S = \left( { - \sqrt 2 ;\sqrt 2 } \right)\)
    • Câu 3:
      Giải bất phương trình \({5^{x + 2}} - {2^{x + 4}} > {5^{x + 1}} - {2^{x + 2}} + {2^{x + 3}}.\)
      • A. x>0
      • B. x<0
      • C. x>1
      • D. x<1
    • Câu 4:
      Tìm tập nghiệm S của bất phương trình \(2{\log _3}\left( {4x - 3} \right) + {\log _{\frac{1}{3}}}\left( {2x + 3} \right) \le 2.\)
      • A. \(S = \left( {\frac{3}{4}; + \infty } \right)\)
      • B. \(S = \left[ {\frac{3}{4};3} \right]\)
      • C. \(S =\left( {\frac{3}{4};3} \right]\)
      • D. \(S = \left[ {\frac{3}{4}; + \infty } \right)\)
    • Câu 5:
      Giải bất phương trình \({\log _{\frac{1}{2}}}^2x + 3{\log _{\frac{1}{2}}}x + 2 \le 0\).
      • A. \(2\leq x\leq 4\)
      • B. \(x\leq 4\)
      • C. \(x\geq 2\)
      • D. \(x \le 2\) hoặc \(x \geq 4\)
    • Câu 6:
      Tìm tập nghiệm S của bất phương trình \({6^{2x + 3}} < {2^{4x - 5}}{.3^{4x - 5}}\).
      • A. \(S = \mathbb{R}\backslash \left\{ 0 \right\}\)
      • B. \(S = \left( { - \infty ;4} \right)\backslash \left\{ 0 \right\}\)
      • C. \(S = \left( {4; + \infty } \right)\)
      • D. \(S = \left( { - \infty ;4} \right)\)
    • Câu 7:
      Giải bất phương trình \({\left( {\sqrt[3]{x} + 1} \right)^5} + \sqrt[3]{x}{.2^{x - 1}} \ge 1.\)
      • A. \(x\geq 1\)
      • B. \(x\leq 1\)
      • C. \(x\geq 0\)
      • D. \(x\leq 0\)
    • Câu 8:
      Tìm tập nghiệm S của bất phương trình \({\log _3}\sqrt {{x^2} - 5x + 6} + {\log _{\frac{1}{3}}}\sqrt {x - 2} > \frac{1}{2}{\log _{\frac{1}{3}}}\left( {x + 3} \right).\)
      • A. \(S = \left( {3;\sqrt {10} } \right)\)
      • B. \(S = \left( {3; + \infty } \right)\)
      • C. \(S = \left( {3;9}\right)\)
      • D. \(S = \left( {\sqrt {10} ; + \infty } \right)\)
    • Câu 9:
      Giải bất phương trình \(x + {\log _3}\left( {x + 1} \right) > 3.\)
      • A. x>-1
      • B. x>-2
      • C. x>2
      • D. x>0
    • Câu 10:
      Tìm tập nghiệm S của bất phương trình \({\log _2}\left( {1 + {3^x}} \right) + {\log _{\left( {1 + {3^x}} \right)}}2 - 2 > 0\).
      • A. \(S=\left( {0, + \infty } \right)\)
      • B. \(S=\left( { - \infty ,0} \right)\)
      • C. \(S=\mathbb{R}\backslash \left\{ 0 \right\}\)
      • D. \(S=\mathbb{R}\)

    Hướng dẫn giải:

    Câu 1:
    \({9^x} - {\log _2}8 < {2.3^x} \Leftrightarrow {\left( {{3^x}} \right)^2} - {2.3^x} - 3 < 0\)

    Đặt: \(t=3^x,t>0.\) Bất phương trình trở thành:

    \(3{t^2} - 2t - 3 < 0 \Leftrightarrow - 1 < t < 3\)

    Kết hợp với điều kiện: \(0 < t < 3 \Rightarrow {3^x} < 3 \Leftrightarrow x < 1.\)

    Câu 2:
    \({2^{{x^2} - x + 1}} > {4^{x + 1}} \Leftrightarrow {2^{{x^2} - x + 1}} > {\left( {{2^2}} \right)^{x + 1}} \Leftrightarrow {2^{{x^2} - x + 1}} > {2^{2x + 2}}\)

    \(\Leftrightarrow {x^2} - x + 1 > 2x + 2 \Leftrightarrow {x^2} - 3x - 1 > 0 \Leftrightarrow \left[ {\begin{array}{*{20}{c}} {x > \frac{{3 + \sqrt {13} }}{2}}\\ {x < \frac{{3 - \sqrt {13} }}{2}} \end{array}} \right.\).

    Câu 3:
    \(\begin{array}{l} {5^{x + 2}} - {2^{x + 4}} > {5^{x + 1}} - {2^{x + 2}} + {2^{x + 3}}\\ \Leftrightarrow {5^x}(25 - 5) > {2^x}({2^4} - {2^2} + {2^3}) \Leftrightarrow {\left( {\frac{5}{2}} \right)^x} > 1 \Leftrightarrow x > 0 \end{array}.\)

    Câu 4:
    ĐK: \(x>\frac{3}{4}\)

    \(\begin{array}{l} 2{\log _3}\left( {4x - 3} \right) + {\log _{\frac{1}{3}}}\left( {2x + 3} \right) \le 2\\ \Leftrightarrow {\log _3}{(4x - 3)^2} - {\log _3}(2x + 3) \le {\log _3}9\\ \Leftrightarrow {\log _3}\frac{{16{x^2} - 24x + 9}}{{2x + 3}} \le {\log _3}9\\ \Leftrightarrow \frac{{16{x^2} - 24x + 9}}{{2x + 3}} \le 9\\ \Leftrightarrow \frac{{16{x^2} - 42x - 18}}{{2x + 3}} \le 0\\ \Rightarrow 8{x^2} - 21x - 9 \le 0 \Leftrightarrow - \frac{3}{8} \le x \le 3 \end{array}\)

    Kết hợp điều kiện: \(\frac{3}{4} < x \le 3.\)

    Câu 5:
    Điều kiện: x>0. Khi đó:

    \(\log _{\frac{1}{2}}^2x = 3{\log _{\frac{1}{2}}}x + 2 \le 0 \Leftrightarrow \left( {{{\log }_{{2^{ - 1}}}}x + 1} \right)\left( {{{\log }_{{2^{ - 1}}}}x + 2} \right) \le 0\)

    \(\begin{array}{l} \Leftrightarrow (1 - {\log _2}x)(2 - {\log _2}x) \le 0 \Leftrightarrow 1 \le {\log _2}x \le 2\\ \Leftrightarrow {2^1} \le x \le {2^4} \Leftrightarrow 2 \le x \le 4 \end{array}\)

    Kết hợp với điều kiện, nghiệm bất phương trình là: \(2\leq x\leq 4.\)

    Câu 6:
    Ta có:

    \(\begin{array}{l} {6^{2x + 3}} < {2^{4x - 5}}{.3^{4x - 5}}\\ \Leftrightarrow {6^{2x + 3}} < {6^{4x - 5}}\\ \Leftrightarrow 2x + 3 < 4x - 5\\ \Leftrightarrow x > 4 \end{array}\)

    Câu 7:
    \({\left( {\sqrt[3]{x} + 1} \right)^5} + \sqrt[3]{x}{.2^{x - 1}} \ge 1\,(1)\)

    Với x<0 thì \(\sqrt[3]{x} < 0;\,\,\,{2^{x - 1}} > 0 \Rightarrow {\left( {\sqrt[3]{x} + 1} \right)^5} < 1;\,\,\sqrt[3]{x}{.2^{x - 1}} < 0.\)

    Do đó VT(1)<1. Vậy bất phương trình không có nghiệm trong khoảng \(\left( { - \infty ;0} \right).\)

    Với \(x\geq 0\) thì \(\sqrt[3]{x} \ge 0;\,\,\,{2^{x - 1}} > 0 \Rightarrow {\left( {\sqrt[3]{x} + 1} \right)^5} \ge 1;\,\,\sqrt[3]{x}{.2^{x - 1}} \ge 0.\)

    Do đó \(VT\,(1) \ge 1\). Vậy bất phương trình có nghiệm \(x \ge 0.\)

    Câu 8:
    Điều kiện: \(\left\{ \begin{array}{l} x - 2 > 0\\ {x^2} - 5x + 6 > 0 \end{array} \right. \Leftrightarrow \left\{ \begin{array}{l} x - 2 > 0\\ (x - 2)(x - 3) > 0 \end{array} \right. \Leftrightarrow x > 3.\) Khi đó:

    \({\log _3}\sqrt {(x - 3)(x - 2)} - {\log _3}\sqrt {x - 2} > - {\log _3}\sqrt {x + 3}\)

    \(\begin{array}{l} \Leftrightarrow {\log _3}\frac{{\sqrt {(x - 3)(x - 2)} }}{{\sqrt {x - 2} }} + {\log _3}\sqrt {x + 3} > 0\\ \Leftrightarrow {\log _3}\sqrt {x - 3} + {\log _3}\sqrt {x + 3} > 0 \end{array}\)

    \(\Leftrightarrow {\log _3}\sqrt {{x^2} - 9} > 0 \Leftrightarrow \sqrt {{x^2} - 9} > 1 \Leftrightarrow {x^2} > 10 \Leftrightarrow x > \sqrt {10}\)

    Vậy tập nghiệm bất phương trình là: \(S = \left( {\sqrt {10} ; + \infty } \right).\)

    Câu 9:
    ĐK: x>-1.

    Xét hàm số \(f(x) = x + {\log _3}(x + 1)\) trên \(\left( { - 1; + \infty } \right).\)

    Ta có \(f'(x) = 1 + \frac{1}{{(x + 1)\ln 3}} > 0\)

    \(\Rightarrow f(x)\) đồng biến trên \(\left( { - 1; + \infty } \right).\)

    Mặt khác \(f(2) = 3\)

    Do đó: \(f(x) > 3 \Rightarrow f(x) > f(2) \Rightarrow x > 2.\)

    Câu 10:
    \(\begin{array}{l} {\log _2}\left( {1 + {3^x}} \right) + {\log _{\left( {1 + {3^x}} \right)}}2 - 2 > 0\\ \Leftrightarrow {\log _2}\left( {1 + {3^x}} \right) + \frac{1}{{{{\log }_2}\left( {1 + {3^x}} \right)}} - 2 > 0\\ \Leftrightarrow {\log _2}^2\left( {1 + {3^x}} \right) - 2{\log _2}\left( {1 + {3^x}} \right) + 1 > 0\\ \Leftrightarrow {\left[ {{{\log }_2}\left( {1 + {3^x}} \right) - 1} \right]^2} > 0\\ \Leftrightarrow {\log _2}\left( {1 + {3^x}} \right) - 1 \ne 0\\ \Leftrightarrow 1 + {3^x} \ne 2 \Leftrightarrow x \ne 0 \end{array}\)